You are on page 1of 12

rt

48+ GMAT MATH QUIZ #2


Ever wondered what it feels like to take the GMAT and score in the top 10% on the quantitative section? Below is a 37-
question math workout to give you the experience of the kinds of questions you are likely to see if you ace the math portion
of the test. If you are aiming for a perfect score, this is a great opportunity to practice your pacing and accuracy, as well as to
work out on the kind of questions ETS is throwing at its top scorers these days. If you haven’t reached the top level yet, this
is a great way to stretch yourself and to practice teasing your brain on the very hardest questions out there.

2 4. What does (1 + m)(1 – m + m2 – m3 ) = ?


1. A fruit salad contains blueberries and the rest
3
raspberries. Chen loves raspberries, so she added (1) m6 = 64
12 quarts of raspberries to the salad. If the 1
5 (2) m -3 = −
mixture is now raspberries, how many quarts 8
7
of fruit salad were there to begin with?
(A) 9 5. If P = (n)(n – 1)(n – 2) . . . (1) and n > 2, what
(B) 12 is the largest value of integer n where P has zero
(C) 15 as its last 6 digits and a non-zero digit for its
(D) 21 millions place?
(E) 25 (A) 29
(B) 30
2x (C) 34
2. What is the value of ? (D) 35
8y (E) 39
(1) x = 3 y + 8
(2) y = 4
6. John is choosing a number n randomly from all
integers from 56 to 150 inclusive. What is the
3. Susan is looking at her cell phone bill from probability that the number he chooses will be
the last 6 months. April’s bill was four times that one where n(n + 1) is divisible by 5?
of all the other bills. If the median amount due 1
for a month was $32, what was the average (A)
5
amount due in a month during those six months?
19
(A) $32 (B)
(B) $48 95
(C) $53 2
(C)
(D) $80 5
(E) $128 19
(D)
94
3
(E)
5

1
The Princeton Review Management, LLC 2002
7. Is x 3 > y ? 12. If k = 20 x + 50 y and x + y = 1 , is k < 35 ?
(1) 3
x>y (1) y >
1
2
(2) x 2 > y
(2) y > x

8. What is the remainder when 332 is divided by 13. If m < 0 , is n < 0 ?


4?
(A) 0
(B) 1 (1) m = n − 3
(C) 2 (2) m n = k , where k > 0
(D) 3
(E) 4
14. There are 7 types of pizza toppings that Al
can order put on his pizza: anchovies, broccoli,
9. If m = 2 x 5 y 7 z and both 350 and 280 are
extra cheese, pepperoni, eggplant, peppers, and
factors of m , what is the minimum value of xyz ?
pineapple. Al hates the combination of anchovies
(A) 700 and pineapple, but loves any other combination of
(B) 70 toppings. How many different combinations of 3
(C) 24 toppings could Al order that he likes (assuming
(D) 6 that he orders any topping no more than once in
(E) 5 any given combination)?
(A) 70
(B) 60
10. Let M be the maximum value and N be the (C) 50
x2 + y (D) 35
minimum value of the expression . If
y (E) 30
a ≤ x ≤ b and c ≤ y ≤ d , then what is the value of
M – N?
15. Which of the following fractions has a
decimal equivalent that terminates?
(1) c = 3 43
I.
(2) a = -2, b = 5 and d = 7 256
35
II.
11. Olivier is an abstract painter who is working 150
on a series of paintings. If each of these paintings 20
III.
has three identical blue vertical stripes, two 99
identical red vertical stripes and two identical (A) I only
black vertical stripes spaced evenly across a (B) II only
square canvass, how many distinct paintings (C) I and II
could Olivier’s series include? (D) II and III
(A) 5040 (E) I, II, and III
(B) 720
(C) 210
(D) 96
(E) 6

2
The Princeton Review Management, LLC 2002
16. If p is the sum of the integers from 75 to 20. John, Aditi, and Karim are all stamp
150 inclusive and q is the sum of integers from collectors. What is the ratio of the number of
75 to 151 inclusive, which of the following stamps Aditi has to the number of stamps John
has.
1 1
expresses + in terms of q ? (1) Aditi has half as many stamps as John has,
p q and Karim has three times as many stamps
2q − 151 as John has.
(A) 2
q − 151q 1
2q + 151 (2) Aditi has
6
the size collection of what
(B) 2
q + 151q Karim has.
1
(C)
2q + 151
1 21. Chef Gundy is creating a new dessert that
(D) will be made from 3 ingredients. If he has 8
2q − 151
cookies and one flavor of sorbet to choose from,
q − 151
(E) 2 what fraction of the possible arrangements will
q − 151q contain the sorbet?
1
(A)
84
17. If a = 2048 and b = 432 then what is the value 1
1 (B)
of (ab) 3 ? 56
1
(A) 48 (C)
9
(B) 96 1
(C) 216 (D)
(D) 256 3
2
(E) 432 (E)
3
x
18. If x , y , and z are integers, is an integer?
yz 22. If the grocery store is 10 miles west of the
(1) y is a factor of x more than once. mall and 8 miles south of the office park, what is
the approximate distance between the mall and
(2) All of the prime factors of z are also
the office park.
factors of y .
(A) 6
(B) 7
(C) 9
19. Bob, Hans, and Ming are running a race what (D) 11
is the probability that Bob will win the race? (E) 13
(1) The chance of Hans winning is .3.
(2) Seven tenths of the time either Bob or
Ming will win.

3
The Princeton Review Management, LLC 2002
23. Is the distance between exits 4 and 5 longer 26. What is the value of x ?
than 12 miles?
(1) 13x > 12 x
(1) One car drives the distance between the (2) x 2 = 9
two exits in 12 minutes at a rate of 55
miles per hour.
1
(2) Another car takes less than 13 minutes to 27. If 0 < x < , which of the following
2
trave l the distance going 60 miles per expresses the third smallest value?
hour.
(A) x
(B) x 2
24. In a particular college dormitory with 130
people, 57 students watch E. R. and 48 watch (C) x −1
Law & Order. If there are twice as many students 3
(D) x
who watch neither show as who watch both, how 4
many students watch both? 1
(E) x −
(A) 25 2
(B) 35
(C) 45
(D) 50
(E) 70 28.

25.

Which of the following equations best expresses


the line (drawn to scale) above?
The circles above share the same center. If the (A) x = 3 + 3 y
radius of the smaller circle is r and the radius of 1
4 (B) x = 1 − y
the larger circle is that of the smaller one, what 3
3 (C) x = 3 y − 3
is the area of the shaded region, in terms of r ?
1
16 2 (D) x = y + 1
(A) πr 3
3
(E) x = 3 − 3 y
(B) πr 2
7
(C) πr 2
9 29. If rectangle A has width w and length l , and
1 2 w > l , what is the value of w ?
(D) πr
9
1
(E) πr 2 (1) The area of rectangle A = 24
3
(2) The perimeter of rectangle A = 20

4
The Princeton Review Management, LLC 2002
30. Set B has three positive integers with a 35. How many even, three digit integers greater
median of 9. If the largest possible range of the than 700 with distinct, non- zero digits are there?
three numbers is 19, given a certain mean, what is (A) 729
that mean? (B) 243
(A) 22 (C) 108
(B) 10 (D) 88
(C) 9.6 (E) 77
(D) 9
(E) It cannot be determined from the information
given 36. What is the value of x 2 ?

(1) y (2 x + y ) = 32
31. What is the product of positive integers P
and Q? (2) ( x + y ) 2 = 36

(1) 18P + Q = 367


(2) Q < 18 37. The probability that a certain coin will fall
heads up is 50%. What is the probability that it
falls heads up on three of six tosses?
32. If xy ≠ 0 , is x < y ? 41
(A) 6
2
35
(B) 6
(1) x 4 < y 4 2
1
(2) x −3 < y −3 (C) 6
2
21
(D) 5
2
5
33. In sequence P, P4 and P5 are 11 and 9 (E) 4
respectively. Each term after the first two terms 2
in sequence P is either the sum of the previous
two terms if that sum is odd, or half the sum of
the previous two terms if the sum is even. What
is the largest possible product of P1 and P2 ?
(A) 40
(B) 14
(C) 12
(D) 10
(E) 7

34. Is x > y ?

(1) x 2 > y 2
(2) xy < 0

5
The Princeton Review Management, LLC 2002
Answer key:

1.
2.
3.
4.
5.
6.
7.
8.
9.
10.
11.
12.
13.
14.
15.
16.
17.
18.
19.
20.
21.
22.
23.
24.
25.
26.
27.
28.
29.
30.
31.
32.
33.
34.
35.
36.
37.

6
The Princeton Review Management, LLC 2002
32 + 32 + 32 + 32 + 32 + 4(32) 9(32 )
1. This question is a good candidate for a = = 48
6 6
ratio box (convert the fractions talked . The correct answer is (B).
about in the question to ratios). There will
actually be two boxes--the first for the
4. The issue with statement 1 is that you
initial amounts of berries and the second don't know whether m = 2 or -2. But don't
for the final amounts. The boxes look as
stop here. If you plug both values into the
follows:
expression in the question, you will see
Initial Ratio Multiplier Actual that both result in the same value.
Blueberries 2 Statement 2 tells you that m = -2 and is
Raspberries 1 therefore sufficient. The correct answer is
Total 3 (D).

Final Ratio Multiplier Actual 5. This one requires some canny thinking. P
Blueberries 2 is the same as the factorial function.
Raspberries 5 Factorials add a factor of ten, in other
Total 7 words, a zero, every fifth integer. You get
6 zeroes beginning with 30! and a seventh
Then to fill in the actual columns, plug in your zero with 35!, so the largest integer that
answer choices. If you are paying attention, there has only 6 zeros is 34!, so the correct
is an easy way to ballpark the answer choice—the answer is (C).
answers are the initial total amount, so the final
total is the answer choice + 12. That amount 6. When n equals a multiple of 5 or one less
must be divisible by 7. The only answer choice than a multiple of 5, n (n + 1) is divisible
for which that is true is (A), the correct one. You by 5. Two- fifths of all numbers fit that
can test that by plugging it into the boxes. description. Since our range is a round set
of 95 consecutive numbers, the
2. Before you look at the statements, get probability that a number chosen at
2x 2x random from that set will fit the criteria
your base numbers to match: y = 3 y .
8 2 required by the question is 2/5.
To evaluate statement 1, plug in the right
half of the equation from the statement 7. Try plugging in numbers to this yes/no
into the expression in the question: data sufficiency. For statement 1, if x = 8
2x 2 3 y +8 and y = 1, you will get a "yes". On the
3y
= 3 y = 2 3 y +8 − 3 y = 2 8 . Therefore, other hand, if you plug in x = 1/8 and y =
2 2 1/3, you will get a "no", so statement 1 is
statement 1 is sufficient to solve.
not sufficient. For statement 2, the same
Statement 2 gives you no information
first pair of numbers will work, giving a
about x.
"yes", and x = -8 and y = 1 give you a
"no". Therefore, statement 2 is
3. The median is the middle number, and insufficient. Even together the statements
since all but one bill are the same amount, are not sufficient, although the range of
the median is the same as the bill amount numbers that will give you a "no" have
of the majority bills. So there are 5 been significantly decreased. But if you
months of bills at $32 and one month at 4 plug in x = 1/2 and y = 1/6, for example,
x $32. The average can be found this you do get a "no". Therefore, the correct
way: answer is (E). In general, for yes/no
questions that have inequalities with
7
The Princeton Review Management, LLC 2002
exponents such as this one, exceptions can 11. Here we have a permutations question
almost always be found between 0 and 1. with some interchangeable elements. You
should start out by finding the number of
8. Don't panic when you see a question like ways to arrange 7 items: 7!. Then to
this that looks like it involves ridiculous account for the interchangeable items, you
amounts of calculating! There is always a divide by the number of interchangeable
pattern--all you have to do is find it. First items factorial, so the result looks as
of all, you can cross off answers (A), (C), 7 × 6 × 5 × 4 × 3 × 2 ×1
follows: . After you
and (E). For (A) or (E) to be correct, 332 3 × 2 × 1× 2 × 1× 2 × 1
would have to be divisible by 4, but it reduce the fraction, you should get
can't be (do you see why?). And for (C) 7 × 6 × 5 = 210 or (C).
to be correct, 332 would have to be even,
but it won't be (again, can you explain 12. As usual with yes/no questions, you want
why?). Now cycle through the first few to plug in to see what is going on. Here,
exponents of 3 to find the pattern: 30 has a however, the easiest way to see what is
remainder of 1, 31 has a remainder of 3, 32 going on in the question is to start with
has a remainder of 1, and 33 has a statement 1, and rather than plug in a legal
remainder of 3. So the pattern that value for y, find out what happens at the
emerges is that all even exponents of 3 boundary of the inequality. In other
have a remainder of 1 when divided by 4, words, plug in y = 1/2, even though that is
and all odd exponents have a remainder of not allowed by the statement. If you do,
3. Since 32 is even, the correct answer is you will find that x also equals 1/2 and
(B). 1
that k = 35. Therefore, if y > , k > 35 .
2
9. One way of interpreting the question is to
Even if you don't see that k has to be
realize that they are asking you to find the
greater than 35, you know that is either
least common multiple of 350 and 280. going to be greater or less than 35--either
To do that, first find the prime
way, statement 1 is sufficient. Statement
factorization of both numbers:
2 follows a similar logic: pretend for a
350 = 2 × 5 2 × 7 and 280 = 2 3 × 5 × 7 . To moment that x = y . If the two variables
then find the least common multiple, just
are equal, then they both equal 1/2. This
take the largest exponent of each of the
brings us back to the findings of statement
factors of the two numbers and multiply
1, therefore statement 2 must also be
the result: 2 3 × 5 2 × 7 = 1400 . So the correct. This means that the correct
correct answer is (B). answer is (D).

10. In order to find both the maximum values 13. This is a straightforward yes/no data
of the expression, we need to know the sufficiency question. Pick numbers for
maximum of both x and y, found in your variables. In statement 1, if m = -1
statement 2. In order to find the and therefore n = 2, yo u get a "no". If m
minimum, if x could equal 0 (implied by = -5 and n = -2, you get a "yes".
its range covering both negative and Therefore statement 1 is not sufficient. In
positive number), then we know the statement 2, to get a negative base number
minimum of the expression too: no matter to result in a positive number, the
what y equals, the result will be 1. We exponent must be even, but could be
know all of this from statement 2, so the either positive or negative (try m = -1, k =
correct answer is (B). 1, and n = either 2 or -2). Therefore the
statement is insufficient. The problems
8
The Princeton Review Management, LLC 2002
remain when the two statements are 149
combined, so the correct answer is (E). as your target answer. Only answer
150
choice (A) gives you that result.
14. First find the total number of three
topping combos that are possible: 17. Clearly, you don’t want to try to actually
7 ×6×5 do the math here instead rewrite the
= 35 . Now find out how many
3 × 2 ×1 equation as 3 (2048)(432) . Once you see
of those combine anchovies and that you are actually dealing with a cubed
pineapple: A, P, and 5 choices for the root you should try to factor out perfect
third topping makes a total of 5 illegal cubes, this yields
combinations. So how many legal
combinations are there? 35 - 5 = 30 or
3
(8 • 256)(8 • 54) = 3 (8 • 8 • 32)(8 • 54) .
(E). Keep factoring and you will get
3
(8 • 8 • 8 • 4 )(8 • 2 • 27) then combine
15. The key to this question is to factor the those numbers that are not perfect cubes.
denominators. (Don't do the long The answer is (B).
division!) Statement I's denominator is
just 28 . Any exponent of 2, no matter how x
large, can never create a non-terminating 18. For to be an integer, both y and z
yz
decimal. The other two statements have 3
must be proven to be factors of x .
as one of their factors. Since that 3 does
Statement 1 gives you part of that, but is
not cancel into the numerator, the
not on its own sufficient. Statement 2
fractions will never terminate. Therefore
tells you that z is a factor of y , but not
the correct answer is (A).
necessarily of x . Together, however, the
16. First of all, you need to recognize that statements are sufficient. To see this,
1 1 statement one can be summarized as
p = q − 151 . Replace p : + . x
q − 151 q follows: 2 + = an integer. Statement 2
y
From here, the best way to proceed is to
realize that it no longer matters what q can be rewritten as y = zn where n is an
actually equals. Instead, you can plug in integer greater than or equal to 1. The
whatever number you please. If you term zn can substitute for one of the y's in
glance at your answer choices, it should statement 1, and suddenly you have what
become immediately apparent that there the question is asking for. Therefore the
will be a lot of calculation involved if you correct answer is (C).
do not choose your number well, so this is
an occasion where you could break one of 19. Here both statements tell you essentially
the rules for choosing your number and the same thing. Neither one can tell you
set q = 1 --it is okay to do that once you the probability that Bob will win--only the
have hit an advanced level, see that using probability that Hans will win. Therefore
1 will make your life significantly easier, the correct answer is (E).
and check all five answer choices. You
will have to make sure that you don't get 20. Only statement 1 relates Aditi to John, so
more than one answer choice working out, the correct answer is (A).
but the ease of calculation makes the risk
worth taking. If you do plug in 1, you get 21. To answer this question you need to find
out how many combinations of
ingredients contain the sorbet and then
9
The Princeton Review Management, LLC 2002
how many combinations of ingredients r in the answer choices, only (C) gives
there are total. If the dessert contains the you your target answer.
sorbet, then there are 8 types of cookies to
choose from for the last 2 ingredients, so 26. Statement 1 tells you that x is positive, but
8×7 not what it equals. Statement 2 tells you
you get or 28 combinations. To
2 ×1 that x = ±3 . Together you have enough
find all possible combinations, you have 9 information to solve. Therefore, the
9×8×7 correct answer is (C).
items to choose for three slots:
3 × 2 ×1
1
or 84 combinations. The fraction that will 27. Plug in. If you chose x = , then (A) is
28 1 4
contain sorbet then is = or (D).
84 3 1 1 3
, (B) is , (C) is 4, (D) is , and (E)
2 8 16
22. Don't get fooled by the 8 and 10--this is 1
is − . Therefore the correct answer is
not a 6:8:10 triangle! The side length 10 4
is not the hypotenuse. Instead, plug the (D).
two legs of the triangle into the
Pythagorean formula: 64 + 100 = 164. 28. You could figure out what equation
The square root of 164 is approximately expresses and then translate all of the
equal to 13 (132 = 169). The correct answer choices into the slope-intercept
answer is (E). formula, but that would take a long time.
The fastest way to do this question is to do
23. Statement 1 you do not need to calculate. POE by plugging in the points you do
Since they give you the time and the rate, know. When y = 0, x must be positive, so
we can find the distance. Either that that eliminates (C). When x = 0, y must
distance will be longer or shorter than 12 be positive, so that eliminates (A) and (D).
miles, but either way, we will have an Now, the y-intercept is less than the x-
answer to our question. Statement 2, on intercept. In (B), the y- intercept = 3, the
inspection, tells us that the distance is less x- intercept = 1, so that can't be the right
than 13 miles, but that does not tell us answer either. Therefore (E) is the correct
whether or not the distance is less than 12 answer.
miles. Therefore the correct answer to the
question is (A). 29. Statement 1 can be translated into the
equation wl = 24 . Because there are two
24. This question deals with the groups variables and only one equation, it is not
formula: Total = Group 1 + Group 2 - sufficient. For the same reason, statement
Both + Neither. Once you fill in the parts 2 alone is not sufficient: 2 w + 2l = 20 .
of the formula you are given, plug in your Together, however, it is possible to solve.
answers. Answer choice (A), the correct This is not as obvious as it would be for
answer, gives you 130 = 57 + 48 - 25 + two linear equations, because when you
50, which works out correctly. combine the two equations you get a
quadratic (one way of combine the two
25. Plug in. For example, if the radius of the equations results in 0 = w 2 − 10 w + 24 ),
small circle is 3, the radius of the larger so there are two possible values for w --6
one would be 4. The area of the shaded and 4. But only one of these, 6, is
region in that scenario would be allowed, because of the restriction w > l .
16π − 9π = 7π . When you plug 3 in for Therefore, the correct answer is (C).

10
The Princeton Review Management, LLC 2002
30. Plug in your answers here. Don't start and the third term or by summing the two
with (C), because 9.6 is an annoying terms and cutting them in half. Since it is
number to calculate with. Start with (B) impossible to add a positive integer to 11
instead. If the mean is 10 and the median to get 9, we must be adding an integer to
is 9, what would the largest possible range 11 to get 18. Therefore the third term
of the three integers be? To find that, our must equal 7. Your scratch paper should
three integers must fit into the equation now look like this: _ _ 7 11 9. Now,
a+b+c the second term + 7 must equal either 22
= 10 . The median, b , equals 9, or 11. If it equaled 22, the second term
3
so a + c = 21 . The range is defined as would have to be 15, but then there would
c − a , to make c − a as large as possible, be no legal possible value for the first
given that a + c = 21 , we can set a = 1 term, so the two terms must sum to 11.
and c = 20 . That does give us a range of Therefore, the second term is 4. Your
19, so (B) is the correct answer. scratch paper now looks like _ 4 7 11 9.
Then by the same logic used to this point,
the first term equals either 10 (to sum to
31. Statement 1 on its own clearly will not
14) or 3 (to sum to 7). The largest
give us the value of PQ, nor will statement
possible product of the first two terms is
2 on its own. Together, however, since
therefore 40 or answer choice (A).
we know that both are positive integers,
the statements are sufficient. Since Q <
18, the first expression defines P as the 34. Statement 1 does not tell us which
number of times 18 goes into 367 and Q variables are positive and which are
as the remainder. The correct answer is negative and is therefore not sufficient.
(C). Statement 2 tells us one of the terms is
positive and the other nega tive, but not
which one is which. Together, the
32. When you are dealing with inequalities
statements do not resolve the issue, so the
and exponents to two major areas you
correct answer is (E).
need to test for exceptions are negative
numbers and numbers between 0 and 1.
Statement 1 is insufficient, because 35. This question is a royal pain, no question.
negative values for the variables will give The best way to approach it is to split up
you the opposite result as positive the problem, and treat the 700's, 800's and
numbers. In Statement 2, negative 900's separately. To find the number of
numbers retain their sign, but we need to numbers beginning with a 7 that fit our
look closely at how fractions react. If you criteria, we have 7 choices for our tens
plug in x = 2 and y = 1 , that gives you a digit (excluding 7, because no repeats are
allowed, 0, and whichever even number is
1 1
"no". If you plug in x = , y = , you used as the units digit) and 4 choices (all
2 4 non-zero, even digits) for our ones digit,
also get a "no". Therefore statement 2 is giving a total of 28 choices. The 900's
sufficient and the correct answer is (B). work exactly the same way, because its an
odd hundreds digit as well. For the 800's,
33. This question looks really intimidating, we have 7 choices for the tens and only 3
but if you stick with it, it isn't all that hard. for the ones (8 is no longer a choice for
On your scratch paper to begin with, you the ones column). That leaves us with at
should draw the set-up to look something total of 21 legal numbers beginning with
like this: _ _ _ 11 9. You are told that, an 8. Add 28 + 28 + 21 = 77, so the
by the rules of the sequence, the fifth correct answer is (E).
term, 9, is found either by summing 11
11
The Princeton Review Management, LLC 2002
36. Neither statements 1 nor 2 are sufficient
on its own because we have no idea how
large y is, so we can eliminate answers
(A), (D) and (B). If you multiply
statement 1 out, you get 2 xy + y 2 = 32 ,
which, if you note, is most of the square
of a sum (i.e. statement 2):
( x + y ) 2 = x 2 + 2 xy + y 2 = 36 . If you
subtract statement 1 from statement 2, you
get x 2 = 4 , which is what you are looking
for. Therefore, the correct answer is (C).

37. First find the probability that it will rain


on one arrangement of two rainy days and
three sunny days (i.e. it will rain the first
two days and then not rain for the last
three days). This works out as follows:
4 4 6 6 6 2 2 33
× × × × = 5 . Now find
10 10 10 10 10 5
how many ways there are to arrange 2
rainy days and 3 sunny days:
5 × 4 × 3 × 2 ×1
= 10. Now multiply the
2 × 1× 3 × 2 × 1
probability of any one arrangement
happening times the number of
arrangements and you get
2 23 3 23 3 3
× 10 = or answer choice (E).
55 54

12
The Princeton Review Management, LLC 2002

You might also like